In a triangle \(P Q R\), let \(\vec{a}=\overrightarrow{Q R}, \vec{b}=\overrightarrow{R P}\) and ....

PW Solutions
PW Solutions
115 بار بازدید - ماه قبل - In a triangle \(P Q
In a triangle \(P Q R\), let \(\vec{a}=\overrightarrow{Q R}, \vec{b}=\overrightarrow{R P}\) and \(\vec{c}=\overrightarrow{P Q}\). If \(|\vec{a}|=3,|\vec{b}|=4\) and \(\frac{\vec{a} \cdot(\vec{c}-\vec{b})}{\vec{c} \cdot(\vec{a}-\vec{b})}=\frac{|\vec{a}|}{|\vec{a}|+|\vec{b}|}\), sthen the value of \(|\vec{a} \times \vec{b}|^2\) is: 📲PW App Link - bit.ly/YTAI_PWAP 🌐PW Website - www.pw.live/
ماه قبل در تاریخ 1403/05/09 منتشر شده است.
115 بـار بازدید شده
... بیشتر